• UPSC IAS Exam Pattern
  • UPSC IAS Prelims
  • UPSC IAS Mains
  • UPSC IAS Interview
  • UPSC IAS Optionals
  • UPSC Notification
  • UPSC Eligibility Criteria
  • UPSC Online
  • UPSC Admit Card
  • UPSC Results
  • UPSC Cut-Off
  • UPSC Calendar
  • Documents Required for UPSC IAS Exam
  • UPSC IAS Prelims Syllabus
  • General Studies 1
  • General Studies 2
  • General Studies 3
  • General Studies 4
  • UPSC IAS Interview Syllabus
  • UPSC IAS Optional Syllabus

decision making and problem solving for upsc

UPSC Prelims Topic Wise Questions – Decision Making and Problem Solving

Direction (Q. Nos. 1-9) Given below are eight questions. Bach question describes a situation and is followed by four Directions ossible responses. Indicate the response you find most Choose only one response for each question. The esponses will be evaluated based on the level of appropriateness ppropriate. ir the given situation. Please attempt all the questions. There sno penalty for wrong answers for these eight questions.

Table of Contents

Q1. You have been asked to give an explanation for not attending an important official meeting. Your immediate boss who has not informed you about the meeting is now putting pressure on you not to place an allegation against him/her. You would (2011)

(a) send a written reply explaining the fact (b) seek an appointment with the top boss to explain the situation (c) admit your fault to save the situation (d) put the responsibility on the coordinator of the meeting for not informing

Exp. As you have been asked to give an explanation, so it is the best way to give a written reply explaining the fact. Hence, answer is (a).

Q2. A local thug (bad element) has started illegal construction on your vacant plot. He has refused your request to vacate and threatened you of dire consequences in case you do not sell the property at a cheap price to him. You would (2011)

(a) sell the property at a cheap price to him (b) go to the police for necessary action (c) negotiate with the goon to get a higher price

Exp. In such a situation, going to the police for necessary action is the best action that should be taken. Hence, answer is (b)

Q3. You have to accomplish a very important task for your headquarters within the next two days. Suddenly you meet with an accident. Your office insists that you complete the task. You would

(a) for extension of deadline (b )ask for an extension of your ability to fishmay (c) suggest alternate person to headquarters who may do the needful (d) stay away till you recover

Exp. Informing headquarters of your inability to finish the work one time, explaining the reason is the best action. So, answer is (b)

Q4. You are an officer-in-charge for providing basic medical facilities to the survivors of an earthquake affected area. Despite your best possible effort, people put allegations against you for making money out of the funds given for relief. You would (2011)

(a) let an enquiry be set up to look into the matter (b) ask your senior to appoint some other person in your place (c) not pay attention to allegations (d) stop undertaking any initiative till the matter is resolved

Exp. As you are doing your work honestly and with complete on dedication, so thet best action is not to pay attention to the allegations. Hence, answer is (c).

Q5. You have been made responsible to hire boats at a short notice to be used for an area under flood. On seeing the price mentioned by the boat owners you found that the lowest price was approximately three times more than the approved rate of the government. You would (2011)

(a) reject the proposal and call for a fresh price (b) accept the lowest price (c) refer the matter to the government and wait (d) threaten the boat owners about a possible cancellation of the licence

Exp. Since, the price mentioned by the boat owners are three times the approved government rate so you cannot accept the lowest price at once, as you are a responsible government employee and the money given to you is not In yours. So, in such situation it is best to reject the proposal and call for a fresh price.

Q6. You are the officer-in-charge of a village istor administering distribution of vaccine in an isolated epidemic hit village and you are left with only one vaccine. There is a requirement of that vaccine from the Gram Pradhan and also a poor villager. You are being pressurised by the Gram Pradhan to issue the vaccine to him. You would (2011)

(a) initiate the procedure to expedite the next supply without issuing the vaccine to either (b) arrange vaccine for the poor villager from the distributor of another area (c) ask both to approach a doctor and get an input about the urgency (d) arrange vaccine for the Gram Pradhan from the distributor of another area

Exp. In the given situation, you need to consult a doctor’s advice before reaching to any conclusion. Then, after knowing the need and urgency, you can give medicine. So, answer is (c).

Q7. You have taken up a project to create night-shelters for homeless people during the winter season. Within a week of establishing the shelters, you have received complaints from the residents of the area about the increase in theft cases with a demand to remove the shelters. You would (2011)

(a) ask them to lodge a written complaint in the police station (b) assure residents of an enquiry into the matter (c) ask residents to consider the humanitarian effort made (d) continue with the project and ignore their complaint

Exp. As a responsible project manager, it is your responsibility to pacify the residents, who have complained to you. The best action in such situation is to assure residents of an inquiry into the matter.

Q8. You, as an administrative authority, have been approached, by the daughter-in-law of an influential person regarding harassment by her in-laws on account of insufficient dowry. Her parents are not able to approach you because of social pressures. You would (2011)

(a) call the in-laws for an explanation (b) counsel the lady to adjust, given such a circumstance (c) take action after her parents approach you (d) ask her to lodge a complaint with the police

Exp. Here, the best option is to ask her to lodge a complaint with the police for future actions. Hence, answer is (d). Directions (Q. Nos. 9-16) Given below are seven questions. Each question described a situation responses. Indicate the response you find most appropriate. Choose only one response for each question. The responses will be evaluated based on the level of appropriateness for the given situation. Please attempt all the questions. There is no penalty for wrong answers for these seven questions.

Q9. You have difference of opinion regarding the final report prepared by your subordinate that is to be submitted urgently. The subordinate is justifying the bos information given in the report. You would (2012)

(a) convince the subordinate that he is wrong (b) tell him to reconsider the results (c) revise the report on your own (d) tell him not to justify the mistaken

Exp. Considering the urgency, options (a) and (b) would result in loss of time. Option (c) would result in an unnecessary dispute. So, the best action is to revise the report on your own and after that you can guide subordinate about the report. So, option (c) is best course of action.

Q10. You are competing with your batch-mate for a prestigious award to be decided based on an oral presentation. Ten minutes are allowed for each presentation. You have been asked by the presenon time. Your friend, however, is than the stipulated time period. You would committee to

(a) lodge a complaint to the chairperson against the discrimination (b) not listen to any justification from the committee (c) ask for withdrawal of your name (d) protest and leave the place

Exp. Withdrawal of your name is an escapist attitude. Protesting is correct but lodging a complaint to the concerned authority will give the result. So, option (a) is the best course of action

Q11. You are handling a time-bound project. During the project review meeting, you find that the project is likely to get delayed due to lack of cooperation of the team members. You would (2012)

(a) warn the team members for their non-cooperation (b) look into reasons for non-cooperation (c) ask for the replacement of team members (d) ask for extension of time citing reasons

Exp. Looking into reasons for non-cooperation and then rectifying it is the correct step. Options (c) and (d) show lack of leadership qualities. Option (a) will not solve the problem permanently. So, option (b) is correct.

Q12. You are the chairperson of a state sports committee. You have received a complaint and later it was found that an athlete in the junior age category who has won a medal has crossed the age criteria by 5 days. You would (2012)

(a) ask the screening committee for a clarification (b) ask the athlete to return the medal (c) ask the athlete to get an affidavit from the court declaring his/her age (d) ask the members of the sports committee for their views

Exp. Since, it is proved that athlete has crossed the age criteria so options (a) and (d) are ruled out. Asking for an affidavit does not change the fact. So, best course of action would be to ask the athlete to return the medal. So, option (b) is correct.

Q13. You are handling a priority project and have been meeting all the deadlines and are therefore planning your leave during the project. Your immediate boss 10 does not grant your leave citing the urgency of the (2012) project. You would

(a) proceed on leave without waiting for the sanction (b) pretend to be sick and take leave (c) approach higher authority to reconsider the leave application (d) tell the boss that it is not justified

Exp. Option (a) shows your adamant attitude. Option (b) is ethically not permitted. Arguing with the immediate boss is not correct as it will not bring any result. Option (c) is the best course of action.

Q14. You are involved in setting up a water supply proje in a remote area. Full recovery of cost is impossib in any case. The income levels in the area are lo and 25% of the population is Below Poverty Lin (BPL). When a decision has to be taken on

(a) recommend that the supply of water be free of charge f you would all respects (2012 (b) recommendon a one time fixed sum t the installation of taps and the usage of water be free (c) recommend that a fixed monthly charge be levied onl on the non-BPL families and for BPL families wal should be free (d) recommend that the users pay a charge based on the consumption of water with differentiated charges for non-BPL families

Exp. Options (a) and (b) would result into wastage of water an loss for the project. Options (c) and (d) are good course action but differentiated charges according to income leve and consumption would be a better solution. So, option (d is correct.

Q15. As a citizen you have some work with a governmen department. The official calls you again and again and without directly asking you, sends out feelers for a bribe. You want to get your work done. You would

(a) give a bribe (b) behave as if you have not understood the feelers and persist with your application (c) go to the higher officer for help and verbally- complaining about feelers (d) send in a formal complaint

Exp. Option (a) is not correct and ethically not permitted. Option (b) will not solve your problem. Giving a written complaint about the feeler is logically not correct. So, best course of action is to go to the higher officer for help and verbally complaining about feelers.

Q16. Consider the following statements. (2013)

  • A primary group is relatively smaller in size.
  • Intimacy is an essential characteristic of a primary group.
  • A family may be an example of a primary group.

In the light of the above statements, which one of the following is true?

(a) All families are primary groups (b) All primary groups are families (c) A group of smaller size is always a primary group (d) Members of a primary group know each other

Exp. Option (a) is incorrect because statement (3) says that a family may be an example of primary group and not that all families are. (b) is incorrect because we are told by statement (3) that a family may be a primary group and nothing about all primary groups. (c) is also incorrect because Statement (1) says that a primary group is relatively smaller in size and not whether it is actually small or not. (d) is correct. Statement (2) tells us that intimacy is an essential characteristic of a primary group which means that the members of a primary group know each other intimately.

  • Directions (Q. Nos. 17-21) Given below are six questions. Each question describes a situation and is followed by four possible responses. Indicate the response you find most appropriate. Choose only one response for each question. The responses will be evaluated based on the level of appropriateness for the given situation. Please attempt all the questions. There is no penalty for wrong answers for these six questions.

Q17. You are the head of your office. There are certain houses reserved for the allotment to the office staff and you have been given the discretion to do so. A set of rules for the allotment of the houses has been laid down by you and has been made public. Your personal secretary, who is very close to you, comes to you and pleads that as his father is seriously ill, he should be given priority in allotment of a house. The office secretariat that examined the request as per the rules turns down the request and recommends the procedure to be followed according to the rules. You do not want to annoy your personal secretary.(2013)

In such circumstances, what would you do?

(a) Call him over to your room and personally explain why the allotment cannot be done (b) Allot the house to him to win his loyalty (c) Agree with the office note to show that you are not biased and that you do not indulge in favouritism (d) Keep the file with you and not pass any orders

Exp. Here, we have two objectives according to the question, not to annoy the secretary and to follow the rules and the law. Option (a) allows to both follow the rules as well as not to annoy the secretary and so it should be the first choice. Option (c) follows the rules but might disrupt the relationship with the secretary. In any case following the law and not showing any kind of favouritism are imperatives and cannot be compromised for any friendship. Thus (c) is also an acceptable solution. Option (b) is clearly wrong as it shows favouritism and bias and breaks the official rules. Option (d) is also clearly wrong as not doing your job because it might lead to unfavourable results for someone is also not acceptable.

Q18. While travelling in a Delhi-registered commercial taxi from Delhi to an adjacent city (another State), your taxi driver informs you that as he has no permit for running the taxi in that city, he will stop at its Transport Office and pay the prescribed fee of ₹ 40 for a day. While paying the fee at the counter you find that the transport clerk is taking an extra 50 for which no receipt is being given. You are in a hurry for your meeting. In such circumstances, what would you do? (2013)

(a) Go up to the counter and ask the clerk to give back the money which he has illegally take (b) Do not interfere at all as this is a matter between the taxi driver and the tax authorities (c) Take note of the incident and subsequently report the matter to the concerned authorities (d) Treat it as a normal affair and simply forget about it

Exp. Here, again we have two objectives. Firstly, it is our responsibility to report any case of corruption that we come across. Secondly, time is precious in this situation and reaching the destination on time is also important. In this case, we can either give priority to reaching the meeting. since reporting the case later on will not cause any imeversible damage or we can also take care of the matter then and there. Thus, option (c) is correct and option (a) is also acceptable. Options (b) and (d) are clearly wrong because they ignore the case of corruption. Taking cognizance of such matters even if they don’t fall in ones domain is every citizen’s public responsibility.

Q19. A person lives in a far off village which is almost two hours by bus. The villager’s neighbour is a very powerful landlord who is trying to occupy the poor villager’s land by force. You are the District Magistrate and busy in a meeting called by a local Minister. The villager has come all the way, by bus and on foot, to see you and give an application seeking protection from the powerful landlord. The villager keeps on waiting outside the meeting hall for an hour. You come out of the meeting and are rushing to another meeting. The villager follows you to submit his application. What would you do? (2013)\

(a) Tell him to wait for another two hours till you come back from your next meeting

(b) Tell him that the matter is actually to be dealt by a junior officer and that he should give the application to him

(c) Call one of your senior subordinate officers and ask him to solve the villager’s problem (d) Quickly take the application from him, ask him a few relevant questions regarding his problem and then proceed to the meeting

Exp. Here, we have two objectives to achieve at the same time. Firstly, we have to help the villager who has come from far off. Secondly, we have to reach the second meeting on time. Option (d) allows us to achieve both. Option (c) also achieves the same objectives though it means that the villager will have to wait for a little more time. Options (a) and (d) are wrong as, they will increase the troubles of the villager who might not be able to wait longer.

Q20. There is a shortage of sugar in your district, where you are the District Magistrate. The government has ordered that only a maximum amount of 30 kg sugar is to be released for wedding celebrations. A son of your close friend is getting married and your friend requests you to release at least 50 kg sugar for his son’s wedding. He expresses annoyance when you tell him about the government’s restrictions on this matter. He feels that since you are the District Magistrate you can release any amount. You do not want to spoil your friendship with him. In such circumstances, how would you deal with the situation? (2013)

(a) Release the extra amount of sugar which your friend has requested for (b) Refuse your friend the extra amount and strictly follow the rules (c) Show your friend the copy of the government instructions and then persuade ade him to accept the lower amount as prescribed in the rules (d) Advise him to directly apply to the allotting authority and inform him that you do not interfere in this matter

Exp. Here, we have two objectives, to follow the government orders and to not spoil the friendship. Option (d) allows us to achieve both since we are not directly refusing to help. Option (c) is also correct. However, it involves us into the situation by trying to persuade the friend directly. Option (a) is clearly wrong as it doesn’t follow the government orders. Option (b) is also less acceptable as it does not achieve the objective mentioned in the problem, that is not to lose the friendship since it risks annoying him

Q21. You are incharge of implementing the Family Planning Programme in an area, where there is a strong opposition to the present policy. You want to convince the residents of the need for keeping small families. What would be the best way of communicating this message? (2013)

(a) By logically explaining to the residents the need for family planning to improve health and living standards (b) By encouraging late marriages and proper spacing of children (c) By offering incentives for adopting family planning devices (d) By asking people, who have been sterilised or are using contraceptives to directly talk to the residents

Exp. Here, we have convince the villagers to accept the programme while keeping in mind the local sensibilities Option (a) is the right choice as villagers are more likely to be convinced, if they are presented with logic and reasons directly and clearly Option (d) is also correct. People, who have already had experience of the family planning ramme will be able to address the misgivings and programme concerns as well as explain the benefits and changes the programme has brought into their lives to villagers directly. Option (b) might not work. By directly asking the people to change certain things which are personal and might be part of their culture might not go down well with the villagers Option (c) also might not work. Incentives can only work to a certain extent and people might still consider it as an interference in their personal lives.

In case you still have your doubts, contact us on 9811333901.  

For UPSC Prelims Resources,  Click here

For Daily Updates and Study Material:

Join our Telegram Channel –  Edukemy for IAS

  • 1. Learn through Videos –  here
  • 2. Be Exam Ready by Practicing Daily MCQs –  here
  • 3. Daily Newsletter – Get all your Current Affairs Covered –  here
  • 4. Mains Answer Writing Practice –  here

Visit our YouTube Channel –  here

  • UPSC Prelims 2017 – Question 6
  • UPSC Prelims 2015 – Question 48
  • UPSC Prelims 2020 – Question 59
  • UPSC Prelims 2020 – Question 65

' src=

Edukemy Team

Upsc prelims 2023 – question 20, upsc prelims 2019 – question 42, upsc prelims 2013 – question 98, upsc prelims 2023 – question 76, upsc prelims mock test – modern history 1, upsc prelims 2022 – question 23, upsc prelims 2021 – question 94, upsc prelims 2017 – question 25, upsc prelims 2020 – question 57, upsc prelims 2021 – question 58, leave a comment cancel reply.

Save my name, email, and website in this browser for the next time I comment.

Our website uses cookies to improve your experience. By using our services, you agree to our use of cookies Got it

Keep me signed in until I sign out

Forgot your password?

A new password will be emailed to you.

Have received a new password? Login here

decision making and problem solving for upsc

  • Government Exam Articles
  • Decision Making Concept & Questions

Decision Making Concept & Questions

Decision Making Questions are a part of Verbal Reasoning and a common topic for all major Government exams conducted in the country.

Logical analysis and reviewing the questions carefully are the key factors to answer questions based on this topic. The weightage of marks, however from this section varies between 3-5 marks.

In this article, we shall discuss at length the concept of decision making, tips to answer decision making questions and some sample questions for candidates assistance.

To know the detailed syllabus for logical or verbal reasoning and the exams in which this section is included, candidates can visit the logical reasoning page. 

Decision Making Questions and Answers PDF:- Download PDF Here

The Concept of Decision Making 

Under the decision making questions, candidates need to analyse and review the given information and based on the same need to answer the question. 

There two types of questions which may be asked as a part of decision making. These include:

  • Direct Questions: In such questions, a condition may be given and based on the same the final decision has to be made
  • Complex Questions: In such questions, a set of eligibility criteria are given. Candidates need to analyse the eligibility details given and based on it, answer what candidates can qualify/ appear/ register for an exam or a post

With the increased competition, the complex variety of questions have become more common. These type of questions are lengthy but if reviewed carefully, they are easy to answer. 

Candidates can go through the video on decision making to understand the concept better-

decision making and problem solving for upsc

Candidates can also check the Tips to Solve Decision Making Questions in UPSC CSAT at the linked article.

Decision Making Questions & Answers

Given below are sample questions based on decision making. Candidates can practise them and analyse the types of questions and the format in which they may be asked. 

Directions (Q1 – Q2): Refer to the information given below and answer the following questions:

In a school in New Delhi, a panel of three senior teachers has been formed to elect the Head Boy of the school. Thre probable candidates have been selected by the students: Ajay, Veer and Nitin. Each teacher has to vote either in against or for each student. 

The following criteria are known to us about the selection:

  • Exactly two teachers vote for Ajay
  • Exactly one teacher votes for Veer
  • Exactly one teacher votes for Nitin
  • Teacher 1 votes for Ajay 
  • Teacher 2 votes against Ajay and Nitin
  • Teacher 3 votes against Nitin

Q 1. Which of the given statements is definitely true?

  • Teacher 2 votes against Nitin
  • Teacher 3 is in support of Ajay but against Nitin
  • Teacher 1 is against Ajay
  • Teacher 3 is against Veer
  • None of the above

Q 2. Based on the given information, which of the statements is completely false?

  • If Teacher 3 votes against Veer, then Teacher 2 would have voted for Veer
  • Teacher 1 was against making Veer the Head Boy of the school
  • Teacher 3 voted for Ajay
  • Veer was not supported by Teacher 1

Directions (Q3 – Q4): Analyse the information given below about an election campaign strategy and answer the following questions:

You are supposed to prepare a strategy for election campaigning which is going to start in 6 months time. This is to be done at multiple constituencies involving many agencies. There would be a huge amount of logistical support which would be required once the format has been decided. The Minister has called for a meeting to hear your action plan and you need to be prepared for it.

Q 3. What would be the most effective answer?

  • Concentrate on coordinating with the different agencies and having them approve the strategy
  • Concentrate on setting deadlines, targets and checkpoints in order to meet the deadlines.
  • Concentrate on getting an immediate feel for the contents design and layout of the strategy.
  • Concentrate on logistics: how to move people and equipment from one place to another smoothly

Q 4. What would be the least effective answer?

  • Concentrate on logistics: how to move people and equipment from one place to another smoothly.

decision making and problem solving for upsc

Direction Q5: You have been using a certain computer system to perform your role for years and it has proved to be stable and reliable. Recently, you were informed that it is to be updated next month with new functionality and applications. You are concerned about the time it would take to have a trouble free system as the current system took six months to become trouble free. You now need to decide your response to this news. 

Q 5. What would be the least effective answer?

  • Find out all you can about the system and volunteer to be the first to trial run it.
  • Voice your concern to your superior and recommend that all possible upgrades be delayed until all possible issues have been identified and resolved.
  • Ask All other colleagues to run the new systems for errors so that the quality of your work is not compromised, but seek their reviews.
  • Believe that the appropriate checks have been carried out and wait for the introduction of the upgrade so that you can assess its functionality.

Q 6. As a relatively small part of your role, you are responsible for the maintenance of a database of statistical information regarding meteorological data and its link to road traffic levels at the Indian met department. This is drawn upon every three months for analysis, although for the past few years the results have been consistent and predictable and some people have questioned the need for such detailed data. In order to compile the information, you rely on daily input of information from a number of people in different localities. One day, one of these individuals comes to you to request that data be submitted on a weekly rather than daily basis, to save time. Your boss is on holiday and you are left to make the decision. Which of the given options must not be considered by the employee?

  • Inform them that you cannot authorize it until your boss is back from leave.
  • Make the change as it seems more sensible, and inform your boss when he returns.
  • Inform him that there is no chance of any change in the reporting pattern.
  • Ask the other person involved in it and what is their take and tell this to your boss once he comes back.

Q 7. You are new to your department and your boss has handed you a brief for a presentation that covers what you should tell your department about the new IT system that is being introduced. Your boss would have done it himself, but he has annual leave for the next few days. It contains a lot of facts and you are aware that your boss prefers a more factual dry presentation style. However, you are concerned that the audience would find the content dull and you want to make a good first impression on them. What would be the least effective answer?

  • Respect the approach that your superior would have taken and present just the facts, but try to do so quickly so that the audience doesn’t get too bored.
  • Introduce a range of interactive, entertaining elements to the briefing that increase audience engagement with the content prepared by your boss, but might not reflect the style of your boss.
  • Send a brief around in email form prior to the meeting and then engage in a two-way discussion with the audience about the system.
  • Suggest to your boss that he may like to present the content when he returns, as this will ensure that it is delivered in the way he intended.

Q 8. You have been asked by your boss to meet with a representative from another unit to present the findings from your team’s latest piece of research. After giving you the briefing, your boss advises you to ‘tone down’ your ‘usual style’ as she thinks this will work better. You are not quite sure what she meant by this and feel a little offended: your style has always worked well in the past. How would you respond? What would be the most effective answer?

  • Make your boss understand that your tone is good for the presentation.
  • Ask your boss for more detail on what they meant by the comment.
  • Ask your colleagues what they meant by the comment.
  • Self-reflect upon what your boss meant then change your behaviour accordingly.

Q 9. The public sector where you are working for is to be included for a major restructure. Some posts will vanish, some roles will change and there will be some new opportunities created. The announcement has created some concern within the PSU, especially as it has been performing well up to this point. Your CMD has arranged for a one on one meeting with everyone to discuss the restructuring. You need to decide how best to prepare for the meeting. Which of the given statements must definitely be followed with reference to the information?

  • Spend time planning how accurately to convey your strengths so that your position is safe within the firm.
  • Plan to ask questions about what new opportunities are available and how you can position yourself to take advantage of it.
  • Aim to get as much information from your CMD about the restructure: timescale and impact.
  • Prepare a convincing case concerning why the company should not be included in the restructuring.

Q 10. You work in an office where the work being carried out in different departments is visible. You have noticed that an engineering trainee in another department spends most of the time ‘lost’ with little work to do. You have already brought this issue to the notice of your head of the department but nothing seems to have happened to solve the situation. In addition, the boss is often out of office so does not see it himself. What would be the least effective answer?

  • Go over to the person in question and set them tasks to complete for your department.
  • Speak to someone else in the department and ask them if there is any work the person could complete for them.
  • Speak to the person in question and tell him to find some work to do as they are currently giving a bad impression.
  • Schedule a meeting for tomorrow with the person’s head of the department where you can suggest some work for him.

Q 11. You have sent a survey to multiple units in order to gauge levels of job satisfaction as part of a wide internal survey across the organization. When the results arrive back, you see that by far the biggest cause of dissatisfaction seems to be the pay levels. However, your manager has suggested that in reality staff are content with their pay: this survey is just seen as an opportunity to ask for more. There is no time or budget to repeat the survey and you need to decide how to proceed. What would be the most effective answer?

  • Conduct some online research into other surveys of this nature to see whether this is a common pattern: include this observation in the report and recommendations.
  • Accept your manager’s views and highlight this in your summary report and recommendations
  • Accept the original data and avoid including your manager’s interpretation when collating the summary report and recommendations.
  • Declare the results of the survey around pay levels to be ‘ambiguous’ and instead concentrate on the second most important area of the survey.

Q 12. A new director was recently appointed to manage your directorate after being managed by the previous person for a couple of years. In her introductory speech, the new director states that there will be a number of changes to be made now she has arrived: some will happen quickly and others will be introduced over the next few months. She does not give more detail than this but reassures everyone that people will be informed as soon as possible about the changes. Which of the following is the best step that any employee working under her must take?

  • Request a meeting straight away with the new director and say that you need to know more about the planned changes.
  • Wait a few weeks and then request a meeting with the new director and find out more about the planned changes.
  • Be alert to the news about the changes, but continue to work as usual until more is announced.
  • Try to find out more about the changes that the new director made in other parts of the organization in order to gain an insight into what she might be planning for your area.

Q 13. You are working on a complex project when one of the members of your department gives you some negative feedback about your way of working that comes as a complete surprise to you. You have never received feedback on this area before and you had previously considered it a moderate strength of yours. It has been playing on your mind for the past week as you are keen to be seen in a positive way by others. Choose the most effective answer.

  • Try to improve your image in the eyes of the person who gave you the feedback by describing to them why you acted in the way you did and how it has brought you success in the past.
  • Ask the person who provided the original feedback for more detail about why they think the way they do.
  • Ask a selection of other people for feedback on the area concerned to see if this corroborates the original negative feedback.
  • Self-reflect some more and read up on how to develop in the area identified.

Q 14. You are in the middle of speaking to an audience about some work that had a successful outcome and that you are very proud of about ten minutes into your speech, you notice that a couple of the audience members are yawning and looking at their watches. Choose the most effective answer.

  • Pause at an appropriate point and ask for audience feedback on what you have covered so far.
  • Look directly at them and ask ‘am I keeping you awake?’ in order to embarrass them and make them pay more attention.
  • Speed up your pace of talking and vary your tone in order to re-engage your interest.
  • Make greater use of visual aids and props in order to stimulate their interest.

Q 15. You have been given two weeks in which to review a large body of data, spot certain types of errors and inconsistencies and then produce a corrected and improved version as a result. Your Superior has asked to meet with you after four days to review your progress. Three days into your review you realize that there are also several other types of errors and inconsistencies in the data and you will not have the time to review it all in two weeks, let alone in time for your meeting with your Superior tomorrow. You feel a more realistic timescale would be four weeks. However, you know that a lot of people are awaiting the outcome of your review and corrections and if it is delayed this will hold up the work of other Units. Pick the least effective answer.

  • Use the two weeks to focus on correcting those types of errors that the original assignment called for and complete the assignment on time.
  • Use the meeting with your Superior to argue for an extension of the deadline so that you can correct the further errors you discovered as well.
  • Start correcting all discovered errors and complete as much as you can in two weeks.
  • Start correcting all discovered errors but arrange to feed the results of your review through in stages to other units so that they can begin work.

Candidates can check the answers to these questions in the PDF given below and check how correctly they have answered all the questions:

For clarity on the strategies to take Reasoning Decision Making Questions in the exam, aspirants should go through the following video-

decision making and problem solving for upsc

Emphasizing on each and every topic under the reasoning ability section will help candidates score more not just in one subject but also increase the overall marks. 

Aspirants can also check the detailed Preparation Strategy for Competitive Exams at the linked article.

For any further updates regarding the upcoming Government exams, candidates can visit BYJU’S and keep themselves updated with the latest exam information, preparation tips and study material. 

Government Exam 2023

Government Exams Related Links

Leave a Comment Cancel reply

Your Mobile number and Email id will not be published. Required fields are marked *

Request OTP on Voice Call

Post My Comment

decision making and problem solving for upsc

Connect with us for Free Preparation

Get access to free crash courses & video lectures for all government exams., register with byju's & download free pdfs, register with byju's & watch live videos.

decision making and problem solving for upsc

  • TRP for UPSC Personality Test
  • Interview Mentorship Programme – 2023
  • Daily News & Analysis
  • Daily Current Affairs Quiz
  • Baba’s Explainer
  • Dedicated TLP Portal
  • 60 Day – Rapid Revision (RaRe) Series – 2024
  • English Magazines
  • Hindi Magazines
  • Yojana & Kurukshetra Gist
  • Gurukul Foundation
  • Gurukul Foundation – Delhi
  • Gurukul Advanced
  • TLP Connect – 2025
  • TLP (+) Plus – 2025
  • Integrated Learning Program (ILP) – 2025
  • MAINS PYQs Mastery
  • TLP Plus – 2024
  • Sociology Foundation Course – 2025
  • Sociology Test Series – 2024 (Coming Soon!)
  • Public Administration FC – 2024
  • Anthropology Foundation Course
  • Anthropology Optional Test Series (Coming Soon!)
  • Geography Optional Foundation Course
  • Geography Optional Test Series – Coming Soon!
  • PSIR Foundation Course
  • PSIR Test Series – Coming Soon
  • KPSC ಪಶುವೈದ್ಯಾಧಿಕಾರಿ (Veterinary Medical Officer – VMO) Exam 2024
  • ‘Mission ಸಂಕಲ್ಪ’ – KPSC Foundation Course
  • ‘Mission ಸಂಕಲ್ಪ’ – KPSC Prelims Crash Course
  • Monthly Magazine
  • Syllabus and Exam Pattern
  • Basic and Detailed Strategy
  • Comprehension Strategy
  • Previous Years CSAT Solved Papers

In the next 30 minutes, we will equip you with the broad strategy to handle CSAT paper 2 while busting some of the myths on which the coaching institutes make money and providing you the basic minimum gyan to crack the exam.

How to know the amount of time needed for preparation?

Which skill areas to work on.

  • Comprehension
  • Interpersonal Skills & Logical, Analytical Reasoning
  • Decision-making
  • General Mental Ability & Data Interpretation

Basic Numeracy

Understanding that the pattern has changed now with CSAT being only a qualifying paper, we have written an article on how CSAT still remains relevant to your preparation  – expected changes in difficulty level of the paper? We expect the difficulty level to go up this year. Scoring even qualifying marks can be an issue for a lot of aspirants.

Also, even though the strategy below applies to the old format (before CSP 2015) of the Prelims examination, we are not removing it. Because, the expert committee constituted by the government for examining the pattern of the Civil Services Examination, can rule in favour of reverting to the old format. 

If you are weak in a particular area of CSAT, the strategy will surely help you.

Do this exercise whenever you have decided to prepare for Paper-2. This period should ideally be at least 3-4 months before the exam.

Take up a UPSC CSAT previous year paper (any one of 2011-2014); close your room; and without any disturbance sit for 2 hours and find out how much you can score without practice.

Now, sit and analyze your shortcomings in those specific areas where you may not have scored well. 

  • If you are scoring less than 100 , phewww!!! There is a lot of work to be done. Start practicing daily for 2-3 hours for the coming 5-6 months and even more infact.
  • If your score is somewhere around 100-130 , then you are weak in some particular areas for e.g. comprehension or basic numeracy OR your speed may be less than required. Know those areas, and work on them on a daily basis for the coming months for getting conceptual clarity and speed.
  • If score is around 130-150 , you may be weak in any one of the areas or your speed of solving paper may be slow. Whatever it is, you need some practice for lets say something close to 2-3 months .
  • If its around 150-170 , you may either be slightly weak in any one area; or you may be making a lot of silly mistakes. It can be slow speed too. Whatever it is, you do not need months of practice. If you can spend 1 month or a little more, with 2-3 hours of practice every day; and you are there.
  • If you have scored around 170-190 , know that you need not study for paper-2. Just 15-20 days before the exam, start solving mocks so that you can increase your speed; improve your accuracy and work out few areas where you may be making 3-4 mistakes regularly.
  • If it is more than 190 , Dude stop wasting your time reading this and prepare Paper-1!!!  😉

It ideally depends on your educational and professional background. This is not to say that only those with a specific background can score high in Paper-2; it is just that we all have been in touch with a certain skill set for a long period of time – which becomes our forte and need to work on the gaps.

For e.g. relatively speaking, engineers are more in touch with Mathematics (equations, calculations etc.) and Data interpretation than a Political Science graduate. Similarly, humanities graduates are more in touch with comprehension skills – reading large amounts of text, verbal reasoning; communication skills etc.

Therefore, no matter what background you are from, you will have an edge on a certain skill area; and at the same time, you will need to work on other skill areas. So, engineers will need to work more on comprehension and interpersonal skills; and a political science graduate will have to work more on Mathematics and Data interpretation.

The UPSC CSAT paper is of Easy to Moderate level. The questions can be solved easily with common sense, basic logic and language skills. But what people generally lack is –

  • Enough Practice

Practice is the key here – which brings both speed and accuracy (amount of practice that you need is mentioned in the section on – what is the right time and energy? ).

Then you need to pick the right book. Here we are mentioning the sections of CSAT exam with some good books which will give you clarity on the topic; and contain enough questions for practice.

  • GS Manual Paper-II by TMH
  • Concise CSAT by Madhukar Bhagat – TMH Publications
  • Cracking the CSAT Paper 2 – Arihant Publications
  • Word Power – Normal Lewis

Section-wise Basic strategy

Comprehension – Use the book Concise CSAT by Madhukar Bhagat – TMH Publications.

To sharpen and improve your comprehension skills, the first thing that you need to do is to ‘read a lot’. Read the same editorials, magazines, articles, columns etc. with more attention. Do not be in a hurry to finish them – take time, otherwise you will lose time and yet not get the correct comprehension.

Work on your vocabulary by referring to pocket dictionary (can be on mobile too ); read the book Word power by Norman Lewis (it is very good for building vocabulary)

Spend time with passages in the TMH book to see in which kind of options you are making mistake. Learn the art of ‘objective reading’.

Solve 20-30 previous year CAT passages too. Solving tough passages will give you additional insights into comprehension abilities; and it becomes easy then to handle relatively easier CSAT passages. As the saying goes, tame an elephant, and a monkey becomes easy .

Refer to the Detailed Strategy section to understand more about the above and scoring high in comprehension.

In the long run try to learn 10 new words a day. These words can be the ones which puzzle you while you go through the Hindu, Economic times,Frontline et al or you can directly pick them from the standard books.

Interpersonal skills including communication skills; Logical reasoning and analytical ability:

You will face questions like the one from CSAT 2014 (we are giving only one here – refer to the detailed strategy for a comprehensive guide on this section)

Given the statement:

“Buses are the cause of more accidents than cars, and trucks cause fewer accidents than buses”, which of the following conclusions can we draw?

(a) There are more buses on the road than trucks.

(b) Car drivers are more careful than bus drivers.

(c) Truck drivers are more skilled than either car or bus drivers.

(d) None of the above 

Answer: (d)

You need not apply too much thought. Just the skill of ‘objective reasoning’ is needed. Unless and until something , the questions in this section are pretty basic in nature.

With some practice from the book GS Manual for Paper 2 – TMH, it is easy to crack. The book covers questions 2 notches above UPSC level. Do not ignore these relatively harder questions. Practicing them makes the actual CSAT exam very easy.

Decision-making and problem-solving:

Use the book Concise CSAT by Madhukar Bhagat or GS Manual TMH .

Solve 40-50 questions to get an idea as to what UPSC demands from you.

Take a choice based on both Ethics and Prudence.

Being blind ethically will cost you marks as we have seen in the past.

For example :

Questions will be like (this is from CSAT 2014):

While travelling in a Delhi-registered commercial taxi from Delhi to an adjacent city (another state), your taxi driver informs you that he has no permit for driving in that city, he will stop at its transport office, pay the prescribed fees of Rs. 40 a day. While paying the fees at the counter, you find that the transport clerk is charging extra Rs. 50 for which no receipt is being given. You are in a hurry for your meeting. In such circumstances, what will you do?
  • Go up to the counter and ask the clerk to give back the money which he has illegally taken.
  • Do not interfere at all as this is a matter between the taxi driver and the tax authorities. 
  • Take note of the incident and subsequently report the matter to the concerned authorities.
  • Treat it as a normal affair and simply forget about it.

It is a situational dilemma. If you act ethically with pragmatism, your answers will be correct. You can choose either (1) or (3).

General mental ability; Data interpretation (charts, graphs, tables, data sufficiency etc. – Class X level):

Refer to the syllabus to know the sub-categories of the topics here. For these refer to GS Manual by TMH – covers the topics comprehensively. Instructions are given at the beginning of each exercise to make you understand. First understand the concept and then the techniques of each kind of sub-category (Let us say blood relations, set theory etc.).

A sample question (CSAT 2014):

Practice is the key here. The more patterns and types you would have practiced, the easier it gets in the exam hall. You will be able to quickly identify the right technique for solving each pattern.

Data Interpretation will be very basic in nature. A sample question (from CSAT 2013):

You need not practice Data Interpretation a lot. Just glance through a few examples. Understand all the types of DIs. Solve 30-40 DI passages. This much practice is sufficient.

As we have already stated in the “Syllabus & Exam Pattern”, this section does not carry a lot of weightage (7-10% questions). For this section, refer to the Cracking CSAT- Arihant Publications or GS Manual TMH. Any one will do. See the kind of questions UPSC has asked over the years and practice accordingly considering the difficulty level. We will be giving you a gist of the Basic Numeracy questions solved from all the years (2011-14).

A group of 630 children is seated in a row for group photo session. Each row contains three children less than the row in front of it. Which one of the following number of rows is not possible?

Solution: d)

It is a question based on sequence and series. You can even apply common logic to solve the question. Even in basic numeracy, there are several sub-categories (as mentioned in the “Syllabus & Exam pattern section” ), make sure you practice several questions from each type for e.g. time & work; clock; ratio and proportions etc.

Refer to Comprehension Strategy now. Click on the tab below 🙂    

                                                                                                       

  • Importance of CSAT

Search now.....

Sign up to receive regular updates.

Email Address

Sign Up Now !

decision making and problem solving for upsc

CSAT Decision Making Questions

Decision making questions describe a situation followed by four possible responses. You have to indicate most appropriate response. There is no penalty of wrong answers, so you must attempt all these questions. These questions are part of CSAT Paper 2.

images/info/study/study02.jpg

CSAT 2011 had 8 such questions, CSAT 2012 had 7 such questions and CSAT 2013 had 6 such questions. The objective of decision making is to judge your character, mindset and personality.

CSAT 2012 Questions

1. You have differences of the opinion regarding the final report prepared by your subordinate that is to be submitted urgently. The subordinate is justifying the information given in the report. You would

  • a) Convince the subordinate that he is wrong
  • b) Tell him to reconsider the results
  • c) Revise the report on your own
  • d) Tell him not to justify the mistake

2. You are competing with your batch-mate for the prestigious award to be decided based on an oral presentation. You have been asked by the committee to finish on time. Your friend however, is allowed more than the stipulated time period.

  • a) Lodge a complaint to chairperson against the discrimination.
  • b) Not listen to any justification from the committee
  • c) Ask for withdrawal of your name
  • d) Protest and leave the place

3. You are handling a time bound project. During the project review meeting, you find that project is likely to get delayed due to lack of cooperation of the team members. You would

  • a) Warn the team members for their non-cooperation
  • b) Look into the reasons for non-cooperation
  • c) Ask for the replacement of the team members
  • d) Ask for the extension of time citing reasons

4. You are a chairperson of a state sports committee. You have received a complaint and later it was found that an athlete in junior age category who has won a medal has crossed the age criteria by 5 days. You would

  • a) Ask the screening committee for a clarification
  • b) Ask the athlete to return the medal
  • c) Ask the athlete to get an affidavit from the court declaring his/her age.
  • d) Ask the members of the committee for their views

5. You are handling a priority project and have been meeting all the deadlines and planning your leave during the project. Your immediate boss does not grant leave citing the urgency of the project. You would

  • a) Proceed on leave without waiting for the sanction
  • b) Pretend to be sick and take leave
  • c) Approach higher authority to reconsider the leave application.
  • d) Tell the boss that it is not justified.

6. You are involved in setting up a water supply project in remote area. Full recovery of cost is impossible in any case. The income levels in the area are low and 25% of the population is below poverty line (BPL). When a decision has to be taken on pricing you would

  • a) Recommended that the supply of water be free of charge in all respects
  • b) Recommended that the users pay a onetime fixed sum for installation of taps and the usage of water be free
  • c) Recommended that a fixed monthly charges be levied on the non-BPL families and for BPL families water should be free
  • d) Recommended that the users pay a charge based on the consumption of water with differentiated charges for Non-BPL and BPL families.

7. As a citizen you have some work with a government department. The official calls you again and again; and without directly asking you, sends out feelers for a bribe. You want to get your work done. You would

  • a) Give a bribe
  • b) Behave as if you have not understood the feelers and persist with your application
  • c) Go to the higher officer for help verbally complaining about feelers.
  • d) Send in a formal complaint

CSAT 2011 Questions

8. You have been asked to give an explanation for not attending an important official meeting. Your immediate boss who has not informed you about the meeting is now putting pressure on you not to place an allegation against him/her. You would

  • a) Send a written reply explaining the fact.
  • b) Seek an appointment with the top boss to explain the situation.
  • c) Admit your fault to save the situation.
  • d) Put the responsibility on the coordinator of the meeting for not informing.

9. A local thug (bad element) has started illegal construction on your vacant plot. He has refused your request to vacate and threatened you of dire consequences in case you do not sell the property at a cheap price to him. You would

  • a) Sell the property at a cheap price to him.
  • b) Go to the police for necessary action.
  • c) Ask for help from your neighbours.
  • d) Negotiate with the goon to get a higher price.

10. You have to accomplish a very important task for your Headquarters within the next two days. Suddenly you meet with an accident. Your office insists that you complete the task. You would

  • a) Ask for an extension of deadline.
  • b) Inform headquarters of your inability to finish on time.
  • c) Suggest alternate person to headquarters who may do the needful.
  • d) Stay away till you recover.

11. You are an officer-in-charge for providing basic medical facilities to the survivors of an earthquake affected area. Despite your best possible effort, people put allegations against you for making money out of the funds given for relief. You would

  • a) Let an enquiry be set up to look into the matter.
  • b) Ask your senior to appoint some other person in your place.
  • c) Not pay attention to allegations.
  • d) Stop undertaking any initiative till the matter is resolved.

12. You have been made responsible to hire boats at a short notice to be used for an area under flood. On seeing the price mentioned by the boat owners you found that the lowest price was approximately three times more than the approved rate of the Government. You would

  • a) Reject the proposal and call for a fresh price.
  • b) Accept the lowest price.
  • c) Refer the matter to the Government and wait.
  • d) Threaten the boat owners about a possible cancellation of the licence.

13. You are the officer-in-charge of a village administering distribution of vaccine in an isolated epidemic hit village, and you are left with only one vaccine. There is a requirement of that vaccine from the Gram Pradhan and also a poor villager. You are being pressurized by the Gram Pradhan to issue the vaccine to him. You would

  • a) Initiate the procedure to expedite the next supply without issuing the vaccine to either.
  • b) Arrange vaccine for the poor villager from the distributor of another area.
  • c) Ask both to approach a doctor and get an input about the urgency
  • d) Arrange vaccine for the Gram Pradhan from the distributor of another area.

14. You have taken up a project to create night-shelters for homeless people during the winter season. Within a week of establishing the shelters, you have received complaints from the residents of the area about the increase in theft cases with a demand to remove the shelters. You would

  • a) Ask them to lodge a written complaint in the police station.
  • b) Assure residents of an enquiry into the matter.
  • c) Ask residents to consider the humanitarian effort made.
  • d) Continue with the project and ignore their complaint.

15. You, as an administrative authority, have been approached, by the daughter-in-law of an influential person regarding harassment by her in-laws on account of insufficient dowry. Her parents are not able to approach you because of social pressures. You would

  • a) Call the in-laws for an explanation.
  • b) Counsel the lady to adjust, given such a circumstance.
  • c) Take action after her parents approach you.
  • d) Ask her to lodge a complaint with the police.

CSAT 2013 Questions

16. You are the head of your office. There are certain houses reserved for the allotment to the office staff and you have been given the discretion to do so. A set of rules for the allotment of the houses has been laid down by you and has been made public. Your personal secretary, who is very close to you, comes to you and pleads that as his father is seriously ill, he should be given priority in allotment of a house. The office secretariat that examined the request as per the rules turns down the request and recommends the procedure to be followed according to the rules. You do not want to annoy your personal secretary in such circumstances, what would you do?

  • (a) Call him over to your room and personally explain why the allotment cannot be done.
  • (b) Allot the house to him to win his loyalty.
  • (c) Agree with the office note to show that you are not biased and that you do not indulge in favoritism.
  • (d) Keep the file with you and not pass any orders.

17. While travelling in a Delhi-registered commercial taxi from Delhi to an adjacent city (another State), your taxi driver informs you that as he has no permit for running the taxi in that city, he will step at it Transport Office and pay the prescribed fee of Rs. forty for a day. While paying the fee at the counter you find that the transport clerk taking an extra fifty rupees for which no receipt is being given. You are in a hurry for your meeting. In such circumstances, what would you do?

  • (a) Go up to the counter and ask the clerk to give bark the money which he has illegally taken.
  • (b) Do not interfere at all as this it a matter between the taxi driver and the tax authorities.
  • (c) Take note of the incident and subsequently report the matter to the concerned authorities
  • (d) Treat it as a normal affair and imply forget about it.

18. A person lives in a far off village which is almost two hours by bus. The villager's neighbor is a very powerful landlord who is trying to occupy the poor villager's land by force. You are the District Magistrate and busy in a meeting called by a local Minister. The villager has in all the way, by bus and on foot, to see you and give an application seeking protection from the powerful landlord. The villager keeps on waiting outside the meeting hall for an hour. You come out of the meeting and are rushing to another meeting. The villager follows you to submit his application. What would you do?

  • (a) Tell him to wait for another two hours till you come back from your next meeting.
  • (b) Tell him that the matter is actually to be dealt by a junior officer and that he should give the application to him.
  • (c) Call one of your senior subordinate officers and ask him to solve the villager's problem.
  • (d) Quickly take the application from him, ask him a few relevant questions regarding his problem and then proceed to the meeting.

19. There is a shortage of sugar in your District where you are the District Magistrate. The Government has ordered that only a maximum amount of 30 kg sugar is to be released for wedding celebrations. A son of your close friend is getting married and your friend requests you to release at least 50 kg sugar for his son's wedding. He expresses annoyance when you tell him about the Government's restrictions on this matter. He feels that since you are the District Magistrate you can release any amount. You do not want to spoil your friendship with him. In such circumstances, how would you deal with the situation?

  • (a) Release the extra amount of sugar which your friend has requested for.
  • (b) Refuse your friend the extra amount and strictly follow the rules.
  • (c) Show your friend the copy of the Government instructions and then persuade him to accept the lower amount as prescribed in the rules.
  • (d) Advise him to directly apply to the allotting authority and inform him that you do not interfere in this matter.

20. You are in-charge of implementing the Family Planning programmes in an area where there is a strong opposition to the present policy. You want to convince the residents of the need for keeping small families. What would be the best way of communicating this message?

  • (a) By logically explaining to the residents the need for family planning to improve the health and living standards.
  • (b) By encouraging late marriages and proper spacing of children.
  • (c) By offering incentives for adopting family planning devices.
  • (d) By asking people who have been sterilized or are using contraceptives to directly talk to the residents.

21. You are a teacher in a University and are setting a question paper on a particular subject. One of your colleagues, whose son is preparing for the examination on that subject, comes to you and informs you that it is his son's last chance to pass that examination and whether you could help him by indicating what questions are going to be in the examination. In the past, your colleague had helped you in another matter. Your colleague informs you that his son will suffer from depression if he fails in this examination. In such circumstances, what would you do?

  • (a) In view of the help he had given you, extend your help to him.
  • (b) Regret that you cannot be of any help to him.
  • (c) Explain to your colleague that this would be violating the trust of the University authorities and you are not in a position to help him.
  • (d) Report the conduct of your colleague to the higher authorities.

Home

  • Study Notes
  • Current Affairs
  • Getting Started

NEW!   The Gist (JULY-24)  | E-BOOKS

HOT!   UPSC IAS COMPLETE PDF NOTES  

decision making and problem solving for upsc

(Paper) CSAT: Problem Solving and Decision Making

Problem solving and decision making.

As a civil servant you are expected to make quick, reasoned, logical and welfare oriented decisions. Once the decision is taken you should stick to your stand and should not bow down before political or industry pressure. The questions based on this will test your decision making skills, something, which the prelims never did. You will also be tested on your sincerity, honesty, and integrity towards the civil services values. Till now candidates feigned these qualities in the IAS interview and got through. This will be possible any more.

This part of the examination measures your ability to use logic and common sense in solving of problems confronted in modern day today administration. The questions will generally relate to situations that require you either to take some action, to explain why an action has or would be taken or to interpret what the action implies. In Problem Solving and Decision Making there is a classic five step approach which would enable you to solve most the questions easily. The steps are:

  • Indentify the problem
  • Indentify the possible solutions
  • Select the best solution
  • Eliminate other solutions
  • Make the decision

Now let us look at these steps in greater detail-

1. Identify the problem : Read the question carefully, and clearly identify what is being asked. What is special problem or point of the question? Is there any specific order you must follow? Are there any special instructions you must follow? 2. Indentify the possible solutions : Read all the choices carefully to identify one that best solves the problem. 3. Select the best solution and then defend it : Answer the question why does this solve the problem? Does it solve the problem with least risk, danger or damage. Always keep in mind that the consequences could be physical or mental. Will it solve the problem within the reasonable time? Have you answered all the parts of the question? Are they in correct order? 4. Eliminate other choices that :

  • Are contradictory to what is required
  • Call for unnecessary actions or risk
  • Require that an instruction be disobeyed
  • Insult, disregard or interfere with the basic values of our constitution
  • Benefit only specific person or group
  • Only partially solve the problem
  • Procrastinate the real problem
  • Call for an action you are unauthorised to take are not in a position to implement

5. Make the decision and record it.

Sample Question

Q. You are the District Collector of XYZ district of India. One morning you reach the office and while you were going through some files, you hear some noise of conflict from out side. You call the assistant and enquire about the issue. The assistant tells you that a communal riot has broken out in the city. What would be your immediate course of action?

Analyses the following options and choose your answer using the codes given below-

(a) You will rush to the scene of the conflict immediately and call the leaders of the both the communities to calm the mob. (b) You will instruct the SP of the district to look into the matter. (c) You will inform the Divisional Commissioner and wait for his instructions. (d) You will summon the SP and other senior officials for conference to take the stock of situation and to take necessary action.

  • English Language Comprehension Skills
  • Data Interpretation & Data Sufficiency
  • English Grammar and Usage
  • Logical Reasoning and Analytical Ability
  • Basic Numeracy & Mental Ability

  [+] Find Out More Books

  [+] CSAT Articles  

  • Iasguru's blog

NEW!   UPSC IAS COMPLETE NOTES

NEW!   UPSC MAINS G.S. 12 Years Solved Papers

UPSC IAS STUDY NOTES

NEW! UPSC Exam Complete Study Notes (PDF Available)

  • ALERT: UPSC IAS, IPS, IFS 2025-2026 Exam...
  • Download UPSC IAS PRELIM (GS+CSAT) Question...
  • (Download) UPSC Toppers Study Notes PDF
  • UPSC Exam Complete Study Notes 2024-2025-...
  • (Download) UPSC, IAS MAINS Exam Previous...
  • Getting Started for UPSC, IAS Exam - FAQ for...
  • UPSC Civil Services PRELIM Exam 2024, 2025...
  • UPSC आईएएस प्रारंभिक परीक्षा पिछले वर्ष के...
  • Download E-Books PDF for UPSC IAS Exams
  • (Date Sheet) UPSC IAS EXAM Calendar 2024
  • New! THE HINDU, YOJANA, PIB PDF
  • New! UPSC PRELIM Papers 2004-2024
  • IAS परीक्षा पेपर in Hindi 2004-2024
  • UPSC Syllabus PDF Download
  • New! IAS MAINS Papers 2010-2023
  • PDF Study Notes for UPSC (Hot!)
  • E-books PDF Download  
  • NCERT Books Download  | NCERT Hindi PDF
  • New! UPSC MAINS SOLVED PAPERS PDF
  • OLD NCERT PDF  
  • UPSC 2024 Exam Calendar

UPSC 2024-25  |  Papers  |  PDF Notes  | Coaching  | E-Books

UPSC Hindi  |  यूपीइससी 2024-25   |  पेपर्स  |  IAS HINDI NOTES  

Disclaimer: IAS EXAM PORTAL (UPSC PORTAL) is not associated with Union Public Service Commission, For UPSC official website visit - www.upsc.gov.in

About Us | Contact Us | Terms & Conditions | Privacy Policy

© 2006-2024 IAS EXAM PORTAL - India's Largest Online Community for IAS, Civil Services Aspirants.

  • Civil Services

UPSC IAS Prelims: General Studies Paper II CSAT: Decision Making MCQs Set 1

Decision making questions in the civil services pelims general studies ii csat paper requires practice and the legal orientation of mind. this can be achieved by practicing for the same. here are some practice questions..

Jagran Josh

1.    While travelling in a Haryana registered commercial taxi from Haryana to an adjacent city (another State), your taxi driver informs you that as he has no permit for running the taxi in that city, he will stop at its Transport Office and pay the prescribed fee of Rs.50 for a day. While paying the fee at the counter you find that the transport clerk is taking an extra Rs.50 for which no receipt is being given. You are in a hurry for an office meeting. Under such circumstances, what would you do?

a)    Do not interfere at all as this is a matter between the taxi driver and the tax authorities. b)    Take note of the incident and subsequently report the matter to the concerned authorities. c)    Go up to the counter and ask the clerk to give back the money which he has illegally taken. d)    Treat it as a normal affair and simply forget about it. Answer: b)

Explanation: One should not be late for the office meeting, but it is not wise either to ignore the extra amount illegally taken by the transport office clerk. Therefore, option b) would be appropriate in such circumstance as it will help in reporting any illegal activities to the concerned department and also one can attend the meeting on time. 2.    A person lives in a remote village which is quite far from the city and the bus stop is also three hours away from that village. The villager's landlord is trying to occupy the poor villager's land by force. You are the District Magistrate and busy in a meeting called by a State Minister. The villager has come all the way, by bus and on foot, to see you and give an application seeking protection from the powerful landlord. The villager keeps on waiting outside the meeting hall for an hour. You come out of the meeting and are rushing to another meeting. The villager follows you to submit his application. What would you do?

a)    Tell him that the matter is actually to be dealt by a junior officer and that he should give the application to him. b)    Tell him to wait for another two hours till you come back from your next meeting. c)    Call one of your senior subordinate officers and ask him/her to solve the villager's problem. d)    Quickly take the application from him, ask  few relevant questions regarding his problem and then proceed to the meeting.

Explanation: Attending the meeting is important but it would be harsh not to attend the matter related to the poor villager and listen to his issues. Therefore, it would be wise to call one of the senior subordinate officers and ask him/her to solve the villager's problem. 3.    There is a shortage of wheat in your District where you are the District Magistrate. The Government has ordered that only a maximum amount of 50 kg wheat is to be released for wedding celebrations. A daughter of your close relative is getting married and your  requests you to release at least 100 kg wheat, for his daughter’s wedding. He expresses annoyance when you tell him about the Government's restrictions on this matter. He feels that since you are the District Magistrate you can release any amount at your will. You do not want to spoil your relationship with him. Under such circumstances, how would you deal with the situation?

a)    Release the extra amount of wheat which your relative has requested for. b)    Refuse your relative the extra amount and strictly follow the rules. c)    Show your relative the copy of the Government instructions and then persuade him to accept the amount as prescribed. d)    Advise him to directly apply to the allotting authority and inform him that you do not interfere in this matter.

Explanation: As a District Magistrate, one should adhere to the rules and regulations formulated by the Government. So, it would be wise to show your relative the copy of the Government instructions and then persuade him to accept the lower amount as prescribed in the rules to handle the matter in an intelligent way. 4.    You are a Professor in a University and are setting an examination paper on a particular subject. One of your colleagues, whose daughter is preparing for the examination on that subject, comes to you and informs you that it is his daughter’s last chance to pass that examination and whether you could help him by indicating what questions are going to be asked in the examination. In the past, your colleague had helped you in another matter. Your colleague informs you that his daughter will suffer from depression if she fails in this examination. Under such circumstances, what would you do?

a)    Explain to your colleague that this would be violating the trust of the University authorities and you are not in a position to help him. b)    In view of the help he had given you, extend your help to him. c)    Report the conduct of your colleague to the higher authorities. d)    Regret that you cannot be of any help to him.

Explanation: Under such circumstances, it would be rational to provide an explanation to your colleague that providing information would be violating the trust of the University authorities and you will not be in a position to help him. 5.    You had applied for a new water meter installation in your house located at Rohini, Delhi. It has been almost 3 months but the water meter has not been installed. So, after waiting for 3 months, you try to find out the reasons for non-installation. After making enquiries you find that there was something wrong with the information you filled up in the form. To sort it out you go to the office of Delhi Jal Board and you find out that nothing was wrong with the information. And when you questioned, the clerk replied that it was just a plain oversight. Now how will you respond?

a)    Will ignore it because that’s how things work in government offices. b)    Will report to the head. c)    Will file a written complaint. d)    Will consult a tout outside so that your water meter gets installed quickly. Answer: b)

Explanation: Reporting to the head would be the most appropriate response. If you are not satisfied with the head's response then you may file a written complaint. 6.    Anu has been a very bright student in the class but in the recent examination she has performed very poorly. She is even not behaving well in the class as well as with her friends. You come to know that she is going through a tragic phase as her father died in a car accident recently. The school principal has asked you to call her mother to inform about her poor performance. In such a situation what will you do?

a)    You will ask for re-evaluation of her answer sheet. b)    You will tell her mother about the poor performance. c)    You will tell her mother that Anu’s poor performance is due to the effect of her father’s demise and also counsel her how to help her child to cope with this trauma. d)    You will do nothing.

Explanation: In such a situation, it advisable to inform the parent about the condition of the child and the reason that is causing disturbance to the child. 7.    You are in a recruitment team of a government organisation and have received an anonymous gift of Rs 1 lakh. The recruitment process is going on but you have not got any indication about the sender of this gift. In such a situation, you will:

a)    Take this money and use it for personal purpose. b)    Donate this money to the orphanage. c)    Try to find out the sender and return the money. d)    None of the above

Explanation: In such a situation, you should understand that it may be a bribe that someone is offering you for his/her selection but since the sender is not giving you any indication, that means he is afraid to disclose his identity. Receiving any gift is also a form of bribe. So the best possible option should be to inform your organisation about this. But since this is not an option and the other three options are not suitable., So, the correct option would be (d). 8.    You are working as a Section officer in a government organisation. One day two of your co-workers start fighting over some petty issue creating lots of nuisance in the office. You will:

a)    Take side of one of the co-workers, who is also your good friend. b)    Leave them and let the situation be resolved on its own. c)    Try to mediate and make both of them understand not to fight and create nuisance in the office. d)    Call the police to control the situation.

Explanation: Calling the police is not a solution as it will demean your office's reputation. Also, by taking side of one of the workers will reflect your biased approach. Hence, option (c) is most suitable. 9.    You are working as an officer with a government financial institution. As a team leader of a group you have been sent to a village to identify the grass-root financial problems of the villagers. You will:

a)    Submit your report without consulting other members. b)    Submit a report after consulting all the group members giving all the details and viewpoints of all members. c)    Identify all the problems by proper study. d)    Refrain from doing any work and give all the responsibilities to other members.

Explanation: Submitting the report without consulting other members is against the spirit of group leadership. Also, giving all responsibilities to the other members is against the ethics of leadership. Option (c) seems to be suitable but since it’s a group task hence consulting other group members is necessary. Hence, option (b) is most suitable. 10.    A religious fair is going to happen in Varanasi. As an IPS officer of that area you have been asked to organize a stampede-free religious fair around the holy shrine. You will:

a)    Only allow a certain number of people, which is manageable, to take part in these programmes. b)    Hesitate from taking responsibility and treat it as a burden. c)    Make proper arrangements to control the crowd by deploying police officers at various places. d)    Order the organizers to follow up the security arrangements.

Get here latest School , CBSE and Govt Jobs notification and articles in English and Hindi for Sarkari Naukari , Sarkari Result and Exam Preparation . Download the Jagran Josh Sarkari Naukri App .

  • India Post GDS Merit List 2024
  • India Post GDS Result 2024
  • UP Police Constable Admit Card 2024
  • UGC NET Exam Analysis 2024
  • UPSC Calendar 2025
  • India GDS Merit List 2024 PDF
  • UP Police Exam Analysis 2024 Live Updates
  • National Space Day Speech
  • National Space Day Essay
  • National Space Day Quiz
  • IAS Prelims CSAT Study Material
  • Free IAS Prelims CSAT Decision Making and Problem Solving Ability
  • Free IAS Prelims CSAT Practice Papers
  • Free IAS Prelims CSAT Quizzes
  • IAS Prelims 2021 CSAT

Latest Education News

IBPS Clerk Exam Analysis 2024, August 24: All Shifts Exam Difficulty Level & Good Attempts

UP Police Exam Analysis 2024 Live Updates: Shift 1, 2 UPPRPB Constable Paper Review, Question Asked

UP Police Constable Exam 2024 Live Updates: यहाँ देखें कैसा रहा यूपी पुलिस परीक्षा का पहला दिन, क्या रहा पेपर का कठिनाई स्तर

UP Police Constable Question Paper 2024: यहाँ से डाउनलोड करें 24 अगस्त शिफ्ट 1, 2 के यूपी पुलिस कांस्टेबल प्रश्न पत्र PDF

[OUT] NEET Seat Allotment 2024 LIVE: MCC NEET UG Round 1 Counselling Results Declared at mcc.nic.in

[LINK LIVE] NEET PG Result 2024: NBEMS Releases Merit List at natboard.edu.in, Direct Link to Download Merit List PDF

NEET PG Result 2024 OUT: NBEMS Announces NEET Medical Entrance Exam Results at natboard.edu.in, Download Merit List PDF

Word Search Puzzle: Find the word “also” in 8 seconds!

UOK Result 2024 OUT at uok.ac.in, Direct Link to Download Kota University UG and PG Marksheet

Alagappa University Result 2024 OUT; Direct Link to Download UG and PG Marksheet at alagappauniversity.ac.in

Alagappa University DDE Result 2024 at alagappauniversity.ac.in; Direct Link to Download Marksheet PDF

SGBAU Result 2024 OUT at sgbau.ucanapply.com, Direct Link to Download UG and PG Marksheet

A Complete List of the Countries with the Highest Number of Skyscrapers

RMC Recruitment 2024 for 532 Safai Kamdar Posts, Download Notification

National Space Day: India’s Space Achievements Timeline (1962 – 2024)

BEML Recruitment 2024 Notification for 100 trainee vacancies

How 1,200 NRIs Turned Madhapar into Asia's Richest Village

UP Police Answer Key 2024: तुरंत यहां से डाउनलोड करें शिफ्ट 1 और 2 अनऑफ़िशियल पेपर उत्तर कुंजी PDF

UP Police Constable Question Paper 2024: Download UPPRPB Exam Set Wise PDF (All Shifts)

Optical Illusion: Only puzzle masters can find the farmer’s wife in 7 seconds!

  • Skip to primary navigation
  • Skip to main content
  • Skip to primary sidebar

UPSC Coaching, Study Materials, and Mock Exams

Enroll in ClearIAS UPSC Coaching Join Now Log In

Call us: +91-9605741000

IAS Books For Decision Making and Problem Solving – Prelims Paper 2 (CSAT)

Last updated on November 4, 2020 by Alex Andrews George

Decision Making and Problem Solving test area comprise questions from “Situation Analysis”. There may not be negative marking in these questions.

IAS Books For Decision Making and Problem Solving – Prelims Paper 2 (CSAT)

Recommended IAS Books for Decision Making and Problem Solving

  • CSAT Decision Making Problem Solving & Interpersonal Skills
  • Decision Making And Problem Solving Strategies

Print Friendly, PDF & Email

Top 10 Best-Selling ClearIAS Courses

Upsc prelims cum mains (pcm) gs course: unbeatable batch 2025 (online), rs.75000   rs.29000, upsc prelims marks booster + 2025 (online), rs.19999   rs.14999, upsc prelims test series (pts) 2025 (online), rs.9999   rs.4999, csat course 2025 (online), current affairs course 2025 (online), ncert foundation course (online), essay writing course for upsc cse (online), ethics course for upsc cse (online), upsc interview marks booster course (online), rs.9999   rs.4999.

decision making and problem solving for upsc

About Alex Andrews George

Alex Andrews George is a mentor, author, and social entrepreneur. Alex is the founder of ClearIAS and one of the expert Civil Service Exam Trainers in India.

He is the author of many best-seller books like 'Important Judgments that transformed India' and 'Important Acts that transformed India'.

A trusted mentor and pioneer in online training , Alex's guidance, strategies, study-materials, and mock-exams have helped many aspirants to become IAS, IPS, and IFS officers.

Reader Interactions

decision making and problem solving for upsc

April 18, 2015 at 10:44 pm

I am a ca and wish to pol science as an optional subject. Kindly advicw me is it a good subject and a scoring one. I have special interwst in polity and international afairs, though i have not studied this subject after 10 class in school. Kindly advice, is it a scoring subject?

decision making and problem solving for upsc

January 20, 2019 at 12:17 pm

I think u are confused . As u described that u are interested in polity but this not means that u score well in polity in my opinion u should read 4-5 optional subjects 14 days each then decide. I hope this will helpful to u.

decision making and problem solving for upsc

August 18, 2017 at 11:32 am

Sir please provide an article on judiciary, new development, commercial etc. I thank you

Leave a Reply Cancel reply

Your email address will not be published. Required fields are marked *

Don’t lose out without playing the right game!

Follow the ClearIAS Prelims cum Mains (PCM) Integrated Approach.

Join ClearIAS PCM Course Now

UPSC Online Preparation

  • Union Public Service Commission (UPSC)
  • Indian Administrative Service (IAS)
  • Indian Police Service (IPS)
  • IAS Exam Eligibility
  • UPSC Free Study Materials
  • UPSC Exam Guidance
  • UPSC Prelims Test Series
  • UPSC Syllabus
  • UPSC Online
  • UPSC Prelims
  • UPSC Interview
  • UPSC Toppers
  • UPSC Previous Year Qns
  • UPSC Age Calculator
  • UPSC Calendar 2024
  • About ClearIAS
  • ClearIAS Programs
  • ClearIAS Fee Structure
  • IAS Coaching
  • UPSC Coaching
  • UPSC Online Coaching
  • ClearIAS Blog
  • Important Updates
  • Announcements
  • Book Review
  • ClearIAS App
  • Work with us
  • Advertise with us
  • Privacy Policy
  • Terms and Conditions
  • Talk to Your Mentor

Featured on

ClearIAS Featured in The Hindu

and many more...

ClearIAS Programs: Admissions Open

Thank You 🙌

UPSC CSE 2025: Study Plan

decision making and problem solving for upsc

Subscribe ClearIAS YouTube Channel

ClearIAS YouTube Image

Get free study materials. Don’t miss ClearIAS updates.

Subscribe Now

IAS/IPS/IFS Online Coaching: Target CSE 2025

ClearIAS Course Image

Cover the entire syllabus of UPSC CSE Prelims and Mains systematically.

decision making and problem solving for upsc

  • Exam Preparation
  • Exams by UPSC

Sorry, there was a problem.

Kindle app logo image

Download the free Kindle app and start reading Kindle books instantly on your smartphone, tablet or computer – no Kindle device required .

Read instantly on your browser with Kindle for Web.

Using your mobile phone camera, scan the code below and download the Kindle app.

QR code to download the Kindle App

Image Unavailable

Comprehension, Decision Making & Problem Solving Compendium for UPSC IAS Prelims CSAT Paper 2 & State PSC Exams 3rd Edition

  • To view this video download Flash Player

decision making and problem solving for upsc

Follow the author

Disha Experts

Comprehension, Decision Making & Problem Solving Compendium for UPSC IAS Prelims CSAT Paper 2 & State PSC Exams 3rd Edition Paperback – 14 July 2022

Save extra with 2 offers.

  • Free Delivery

10 days Replacement

  • Amazon Delivered
  • Pay on Delivery
  • Secure transaction
Replacement Reason Replacement Period Replacement Policy
Physical Damage,
Defective,
Wrong and Missing Item
10 days from delivery Replacement

Replacement Instructions

decision making and problem solving for upsc

Purchase options and add-ons

Comprehension, Decision Making & Problem Solving Compendium for IAS Prelims General Studies Paper 2 & State PSC Exams is the 2nd of the 3 books for Paper 2.

• It is an exhaustive work capturing all the important topics being asked in the last few years of the IAS Prelim exam. The book has separate units for Comprehension and English Language Comprehension.

• English Language RC passage covers all literary styles.

• Exhaustive exercise of situation-based questions to test decision making and administrative course of action.

• Vast variety of situation-based questions to test Interpersonal Skills including Communication Skills.

• The book is divided into chapters which contains detailed theory explaining all concepts with proper examples along with Practice Exercise.

• The Exercise covers the fully solved past CSAT questions from 2011 onwards.

In all the book contains 5 Chapters & 1500+ MCQs (Past + Previous) with detailed solutions.

  • ISBN-10 9355641052
  • ISBN-13 978-9355641052
  • Publisher Disha Publication
  • Publication date 14 July 2022
  • Part of series English Edition NCERT based Compendium Series for IAS Prelims (Papers 1 & 2) & Mains
  • Language English
  • Dimensions 20.3 x 25.4 x 4.7 cm
  • Print length 204 pages
  • See all details

Frequently bought together

Comprehension, Decision Making & Problem Solving Compendium for UPSC IAS Prelims CSAT Paper 2 & State PSC Exams 3rd Edition

Products related to this item

Oswaal UPPSC Prelims 15 Year-Wise Solved Papers General Studies Paper-I (2009-2023) General Studies (CSAT) Paper-II (2013-2023) For 2024 Exam

From the brand

Brand Banner

From the Publisher

1

Product description

About the author, product details.

  • Publisher ‏ : ‎ Disha Publication (14 July 2022); AIETS.COM PVT. LTD. Plot No -6 ,(Ground Floor) Near IPSR Institute, Khasra No-40 Satbari, Chattarpur, New Delhi - 110074
  • Language ‏ : ‎ English
  • Paperback ‏ : ‎ 204 pages
  • ISBN-10 ‏ : ‎ 9355641052
  • ISBN-13 ‏ : ‎ 978-9355641052
  • Item Weight ‏ : ‎ 360 g
  • Dimensions ‏ : ‎ 20.3 x 25.4 x 4.7 cm
  • Country of Origin ‏ : ‎ India
  • Net Quantity ‏ : ‎ 1 Count
  • Importer ‏ : ‎ AIETS.COM PVT. LTD. Plot No -6 ,(Ground Floor) Near IPSR Institute, Khasra No-40 Satbari, Chattarpur, New Delhi - 110074
  • Packer ‏ : ‎ AIETS.COM PVT. LTD. Plot No -6 ,(Ground Floor) Near IPSR Institute, Khasra No-40 Satbari, Chattarpur, New Delhi - 110074
  • Generic Name ‏ : ‎ Civil Services
  • #70 in UPSC Civil Services Prelims

About the author

Disha experts.

Disha Experts is a team of the most renowned and prolific content writers pioneering in School and Test Prep segments (Competitive Exams & Government Vacancies). It consists of more than 30 experts comprising of PhDs, B-Techs, PGs, etc. working full-time with decades of experience in various academic and competitive exam verticals. Disha Experts has emerged as a think tank in the School and Test Prep segments providing the most updated (latest pattern & syllabus-wise) to-the-point student-friendly (relevant & easy-to-understand presentation) content such that the aspirants can give their best.

The Economics of World War II: Six Great Powers in International Comparison (Studies in Macroeconomic History)

Customer reviews

  • 5 star 4 star 3 star 2 star 1 star 5 star 58% 24% 6% 0% 12% 58%
  • 5 star 4 star 3 star 2 star 1 star 4 star 58% 24% 6% 0% 12% 24%
  • 5 star 4 star 3 star 2 star 1 star 3 star 58% 24% 6% 0% 12% 6%
  • 5 star 4 star 3 star 2 star 1 star 2 star 58% 24% 6% 0% 12% 0%
  • 5 star 4 star 3 star 2 star 1 star 1 star 58% 24% 6% 0% 12% 12%

Reviews with images

Customer Image

  • Sort reviews by Top reviews Most recent Top reviews

Top reviews from India

There was a problem filtering reviews right now. please try again later..

decision making and problem solving for upsc

  • Press Releases
  • Amazon Science
  • Sell on Amazon
  • Sell under Amazon Accelerator
  • Protect and Build Your Brand
  • Amazon Global Selling
  • Supply to Amazon
  • Become an Affiliate
  • Fulfilment by Amazon
  • Advertise Your Products
  • Amazon Pay on Merchants
  • Your Account
  • Returns Centre
  • Recalls and Product Safety Alerts
  • 100% Purchase Protection
  • Amazon App Download
 
  • Conditions of Use & Sale
  • Privacy Notice
  • Interest-Based Ads

decision making and problem solving for upsc

Test: Decision Making & Problem Solving - UPSC MCQ

15 questions mcq test - test: decision making & problem solving, all, except one of the following are important strategies that a manager can use to create a more effective decision making environment. which one is not.

Encourage others to make decisions

Be ready to try new things

Relying solely upon himself/herself

Recognise the importance of quality information

To get effective decision making environment, manager should encourage others to make decisions as well as should recognise the importance of quality information. Manager is also expected to try new things to improvise the outcome of decision. So, manager should encourage for participative/ democratic process of decision making rather relying solely upon himself/herself.

decision making and problem solving for upsc

Errors in decision making occur because we use decision making heuristics

in an unplanned manner

haphazardly

beyond the range for which they are intended

without taking care of other aspects

  • Heuristics are simple, efficient rules based on experiences which people often use to form judgements and make decisions. This usually involve focusing on one aspect of a complex problem and ignoring others.
  • Although, these rules work well under many circumstances, but they can lead to systematic deviation from logic or rationality resulting errors in decision making.
1 Crore+ students have signed up on EduRev. Have you?

A manager’s steps in a decision making process are given below. Arrange them in correct sequence and choose the correct answer.  1. Define the problem. 2. Identify the limiting factors. 3. Develop potential alternatives. 4. Establish a control and evaluation system.

2, 1, 3, 4 

The correct sequence of a manager's steps in the decision-making process is: 1, 3, 4, 2. 1. Define the problem. 3. Develop potential alternatives. 4. Establish a control and evaluation system. 2. Identify the limiting factors..

A group effort of generating alternative ideas that can help a manager to solve a problem is called

The Delphi technique

Out of the box thinking

The nominal group technique

Brain storming

  • The Delphi technique is used in structured communication technique to reach the correct response through consensus.
  • Out of box thinking generates unconventional ideas.
  • Nominal group technique is the decision making process which prevents the domination of discussion by single person, encourage the more passive group members to participate.
  • In brain storming, individuals work alone and then groups were asked to brainstorm a list of ideas using procedures that encouraged creativity and discouraged criticism and evaluation. The scores of the individuals who worked alone were combined, averaged and compared with the scores of the groups.

A quantitative technique for decision making that shows a complete picture of potential alternative decision paths is called

A decision tree

Pay back analysis

Decision tree, a schematic tree-shaped diagram used to determine a course of action or show a statistical probability. Each branch of the decision tree represents a possible decision or occurrence. The tree structure shows how one choice leads to the next and the use of branches indicates that each option is mutually exclusive. This technique is used as quantitative technique decision making.

In decision making, judging a sample on the basis of similarity and random looking appearance is known as

Representativeness heuristic

Availability heuristic

Anchoring heuristic

Adjustment heuristic

Representativeness heuristic, a mental shortcut that helps us in making a decision by comparing information to our mental prototypes. e.g. if someone was to describe an older woman as warm and caring with a great love of children, most of us would assume that the older woman is a grandmother. She fits our mental representation of a grandmother, so we automatically classify her into that category.

While addressing people in an open ground, it begins to rain. Your address is important and you cannot afford to postpone it. What will you do? You would

ask some one to arrange an umbrella

stop your address for a while

continue the address without bothering the rain

postpone it for a day

Asking some one to arrange an umbrella as mentioned in option (a) will not be appropriate while options (b) and (d) do not serve the purpose as it is mentioned that it is important and you cannot afford to postpone it. Here, in this case continuing the address without bothering the rain as mentioned in option (c) will be the best course of action.

You are head of your office. Some media people come to your office and request you to brief them about the pension plan of your office. You will

aks the media people to come some another day

immediately accept the offer and brief them

get them turned out of your office

ask them to wait and to consult the officials concerned

Here, options (a) and (c) are not appropriate as these are not valid on ethical grounds while option (b) is also not valid as you are the head of the office and you have certain important tasks to accomplish. In this case, option (d) would be the best course of action as there are certain officials for each work in every office, so one has to consult them.

National Human Rights Commission took cognizance of a media report on the death of a lady by the wrong blood transfusion in a government hospital, issued a show cause notice as to why the monetary relief should not be given to the death’s kin. The notice was issued to the special secretary, health and family welfare of a district. Which of the following actions would solve the problem?

Special secretary should get confirmation from the medical superintendent whether it was transfusion reaction that caused the death, if is true the compensation should be paid

The lab technician should be removed from the post for giving wrong blood

Both ‘a’ and ‘b’

Do not reply to the notice given by the commission

Not replying to the Commission may lead to further undesirable situation as the truth is in the media report which made commission take cognizance. Thus, the option (c) will only solve the problem which led to death.

A district library in your town is facing numerous problems. The books are eating dust, it is too noisy and also dirty inside and outside the library. There is no staff to maintain the books or rooms to keep them in safe custody. You are a regular visitor to the library, but have stopped going to the library, because of above problems. Even after some days, when you saw the library it was under the same condition. Now, you will

write a detailed letter to the ministry of culture and ask the ministry to improve the library

not take any action as you know that no concerned authority is going to take care of it

write a letter to the local newspaper to make a detailed report on the problems faced by library

Both ‘a’ and ‘c’

Being a responsible citizen and a regular visitor to the library, you may not stop from taking some proper action to help improve the condition of the library. Informing newspaper reporter to make a report is the best way to tackle the problem besides this it is necessary to write to the concerned authority to take action. Thus, option (d) is the best answer.

As a citizen you have some work with a government department. The official calls you again and again and without directly asking you, sends out feelers for a bribe. You want to get your work done. You would

give a bribe

behave as, if you have not understood the feelers and persist with your application

go to the higher officer for help and verbally complaining about feelers

send in a formal complaint

Option (a) is not correct and ethically not permitted. Option (b) will not solve your problem giving a written complaint about the feeler as mentioned in option (d) is logically not correct. So, best course of action is to go to the higher officer for help and verbally complaining about feelers.

You are competing with your batchmate for a prestigious award to be decided based on an oral presentation. Ten minutes are allowed for each presentation. You have been asked by the committee to finish on time. Your friend, however, is allowed more than the stipulated time period. You would

lodge a complaint to the chairperson against the discrimination

not listen to any justification from the committee

ask for withdrawal of your name

protest and leave the place

Option (b) would not be moral as everyone has the right to give justification while option (c) is an escapist attitude. Protesting is correct, but lodging a complaint to the concerned authority will give the result, so, option (a) is the best course of action.

You are a District Collector and are going to approve a proposal for payment of subsidy under government rules to an industrialist. You approve it as a normal duty. The same industrialist attends a social gathering at your home and gifts you a packet, on opening which you find a costly diamond jewellery set. You are shocked for a few minutes. Which is the best among the following you consider?

Sell the diamond jewellery to get cash

Ask your wife to deposit the diamond jewellery in their bank locker

Keep in the house locker

Return it back to the industrialist

Option (a) shows you made the proposal to get cash in kind and (b) or (c) shows you are money minded and not honest to the work. Only option (d) shows that you are honest and not glutton for money or gifts.

You are a Senior Police Officer. You sent your subordinate on some work by jeep. The driver and the constable in the jeep are unarmed. On the way they happen to notice a jeep in broken condition. A few persons sitting inside it ask for their help. As your subordinate gets down to help those persons, he is attacked from behind by a gang of criminals. They loot him and escape quickly with the persons sitting inside the car. The officer is charged for his negligence, now you as a senior officer, will

suspend him for his lack of common sense

frame charge against him for stopping the jeep mid-way

not hold him responsible for this accidental happening

accuse him of not being precautious

Opting for (a), (b) and (d) are improper. The decisions will be without information and also lack in empathy. Thus, option (c) is the correct one as the incident was an accident and not the deliberate mistake or negligence in the performance of duty at the part of the subordinate.

You are a Police Commissioner and you notice that one of your subordinate was a good officer with good manners, but over a few months his appearances and ways have changed. He has changed his behaviour, looks shabby other officers have also complained about his ill-mannered behaviour. You fear something might be wrong and also worry the effect of his shabby looks that might affect his relationship adversely with his colleagues. How do you approach him?

You will meet him separately and talk at length and try to know the cause of his problem

You will call on him at his chamber and warn about his shabby appearance

Show him sympathy and ask him to seek his transfer

You will send him on leave to help recover himself

Option (a) will be most appropriate answer; option (b) will be negative approach whereas options (c) and (d) are irresponsible and negligence and show that you lack in coordination as a higher officer. So, option (a) suits to be a best option.

--> and get INR 200 additional OFF

Top Courses for UPSC

decision making and problem solving for upsc

Important Questions for Decision Making & Problem Solving

Decision making & problem solving mcqs with answers, online tests for decision making & problem solving.

cation olution
Join the 10M+ students on EduRev

Welcome Back

Create your account for free.

decision making and problem solving for upsc

Forgot Password

IMAGES

  1. Buy Comprehension, Decision Making & Problem Solving Compendium for

    decision making and problem solving for upsc

  2. Buy Comprehension, Decision Making & Problem Solving Compendium for

    decision making and problem solving for upsc

  3. problem solving and decision making competency examples

    decision making and problem solving for upsc

  4. Decision Making and Problem Solving

    decision making and problem solving for upsc

  5. Books for Decision making and Problem solving for UPSC Mains

    decision making and problem solving for upsc

  6. Buy Comprehension, Decision Making & Problem Solving Compendium for

    decision making and problem solving for upsc

COMMENTS

  1. Decision Making and Problem Solving

    Decision Making. Decision making is the process of making choices by evaluating alternatives. It requires analytical and critical thinking skills, alongside an understanding of the implications and consequences of each option. 2. Problem Solving. Problem-solving entails identifying, analyzing, and resolving problems in a systematic manner.

  2. Decision Making and Problem Solving for CSAT

    Decision Making and Problem Solving Questions for CSAT. There can be different types of questions testing your decision-making skills or problem-solving skills. The most common type of question asked is the situation analysis type. Example of a decision making question ( Civil Services Prelims 2012) Qn: You are handling a time-bound project.

  3. UPSC Prelims Topic Wise Questions

    UPSC Prelims Topic Wise Questions - Decision Making and Problem Solving. by Edukemy Team May 4, 2024. Direction (Q. Nos. 1-9) Given below are eight questions. Bach question describes a situation and is followed by four Directions ossible responses. Indicate the response you find most Choose only one response for each question.

  4. How to Solve Decision Making Questions in UPSC CSAT Paper

    2) Pretend to be sick and take leave. 3) Approach higher authority to reconsider the leave application. 4) Tell the boss that it is not justified. Answer: (3) Strategy to solve decision making questions for UPSC exam: Decision-making problems can be solved by a five-step approach:-. Find the issue in the question.

  5. Decision Making and Problem Solving UPSC CSAT: Crash Course (English

    Importance of Decision Making & Problem Solving UPSC: Understanding Decision Making & Problem Solving is crucial for UPSC exam success. Knowing its pattern, syllabus, and question paper analysis can significantly boost preparation. Practice papers and mock tests help improve performance.

  6. CSAT

    Link for Loan Process: https://pluslearner.typeform.com/to/KDACwpj2Enroll Now: https://unacademy.onelink.me/081J/23ffbff2Unacademy Combat on 30th May'21 at ...

  7. Decision Making Concept & Questions

    Decision Making Questions are a part of Verbal Reasoning and a common topic for all major Government exams conducted in the country. Logical analysis and reviewing the questions carefully are the key factors to answer questions based on this topic. The weightage of marks, however from this section varies between 3-5 marks.

  8. Basic Strategy

    The book covers questions 2 notches above UPSC level. Do not ignore these relatively harder questions. Practicing them makes the actual CSAT exam very easy. Decision-making and problem-solving: Use the book Concise CSAT by Madhukar Bhagat or GS Manual TMH. Solve 40-50 questions to get an idea as to what UPSC demands from you.

  9. IAS Books For Civil Services Preliminary Exam Paper 2 ( CSAT)

    This revised edition of Cracking the CSAT Paper - 2: Civil Services Aptitude Test more or less covers the UPSC syllabus, touching areas like verbal and numerical skills, Data Interpretation, General Mental Ability, Interpersonal Skills, and Communication, Decision Making and Problem-Solving Skills, and Analytical Ability. Practice tests ...

  10. Methods of Problem Solving

    This article is significant for UPSC CSE. Discover about Methods of Problem Solving and its various aspects in this article. It is an important topic for many exams. ... delving into the depths of human cognition and decision-making. Learn more about the Types of Research here! Free Mentorship Program by. Ravi Kapoor, Ex-IRS. UPSC Exam-Hacker ...

  11. PDF Decision Making and Problem Solving

    Let's first clarify what we mean by problem solving and decision making and how they go together: •Problem solving is a set of activities designed to analyze a situation systematically and generate, implement, and evaluate solutions. •Decision makings a mechanism for making choices at each step of the problem-solving process. Decision ...

  12. UPSC EDGE for Prelims 2020

    English Grammar | UPSC | IAS | UPSC IAS | English by Akansha Ma'am | UPSC Preparation | CSAT | CSAT English | English CSAT | English Grammar CSAT | CSAT Exam...

  13. CSAT Decision Making Questions

    Decision making questions describe a situation followed by four possible responses. You have to indicate most appropriate response. There is no penalty of wrong answers, so you must attempt all these questions. These questions are part of CSAT Paper 2. CSAT 2011 had 8 such questions, CSAT 2012 had 7 such questions and CSAT 2013 had 6 such ...

  14. Decision Making and Problem Solving (UPSC Previous Years ...

    Decision Making and Problem Solving (UPSC Previous Years' Questions 2011 to 2019) - Free download as PDF File (.pdf), Text File (.txt) or read online for free. This document contains sample questions from previous years' Civil Services Aptitude Tests (CSAT) from 2011-2013 that relate to the theme of decision making and problem solving. Four multiple choice response options are provided for ...

  15. Problem Solving and Decision Making

    In Problem Solving and Decision Making there is a classic five step approach which would enable you to solve most the questions easily. The steps are: Indentify the problem; Indentify the possible solutions; Select the best solution; Eliminate other solutions; Make the decision; Now let us look at these steps in greater detail-1.

  16. Decision Making and Problem Solving

    Importance of Decision Making and Problem Solving UPSC: Understanding Decision Making and Problem Solving is crucial for UPSC exam success. Knowing its pattern, syllabus, and question paper analysis can significantly boost preparation. Practice papers and mock tests help improve performance.

  17. UPSC IAS Prelims: General Studies Paper II CSAT: Decision Making MCQs Set 1

    a) Explain to your colleague that this would be violating the trust of the University authorities and you are not in a position to help him. b) In view of the help he had given you, extend your ...

  18. Decision Making & Problem Solving

    Decision Making & Problem Solving Notes offer in-depth insights into the specific topic to help you master it with ease. This comprehensive document covers all aspects related to Decision Making & Problem Solving. It includes detailed information about the exam syllabus, recommended books, and study materials for a well-rounded preparation.

  19. Problem Solving UPSC Prelims Paper 2 CSAT

    Besides explaining types of Problem Solving - UPSC Prelims Paper 2 CSAT - Quant, Verbal & Decision Making - UPSC - Notes, Videos, & Tests theory, EduRev gives you an ample number of questions to practice Problem Solving - UPSC Prelims Paper 2 CSAT - Quant, Verbal & Decision Making - UPSC - Notes, Videos, & Tests tests, examples and also ...

  20. Interpersonal Skills Including Communication Skills for CSAT

    As of now for solving the Decision-making and problem-solving section, the best answer is often derived by using interpersonal skills too, and that's what UPSC expects from future Civil Servants. How to study Interpersonal Skills for CSAT? We recommend the below sources to learn the subject. Join CSAT Course: UPSC Prelims Paper 2 Program

  21. IAS Books For Decision Making and Problem Solving

    IAS Books For Decision Making and Problem Solving - Prelims Paper 2 (CSAT) Last updated on November 4, 2020 by Alex Andrews George. Decision Making and Problem Solving test area comprise questions from "Situation Analysis". There may not be negative marking in these questions.

  22. Comprehension, Decision Making & Problem Solving Compendium for UPSC

    Amazon.in - Buy Comprehension, Decision Making & Problem Solving Compendium for UPSC IAS Prelims CSAT Paper 2 & State PSC Exams 3rd Edition book online at best prices in India on Amazon.in. Read Comprehension, Decision Making & Problem Solving Compendium for UPSC IAS Prelims CSAT Paper 2 & State PSC Exams 3rd Edition book reviews & author details and more at Amazon.in. Free delivery on ...

  23. UPSC CAPF Syllabus 2024: Check Exam Pattern and Download PDF for ...

    The UPSC CAPF assistant commandant written exam syllabus will consist of two papers, i.e., paper 1 and paper 2. Paper 1 will have 250 marks and will contain questions about general ability and ...

  24. Test: Decision Making & Problem Solving

    Test: Decision Making & Problem Solving for UPSC 2024 is part of UPSC preparation. The Test: Decision Making & Problem Solving questions and answers have been prepared according to the UPSC exam syllabus.The Test: Decision Making & Problem Solving MCQs are made for UPSC 2024 Exam. Find important definitions, questions, notes, meanings, examples, exercises, MCQs and online tests for Test ...